Talk:2014 AMC 12A Problems/Problem 15

Revision as of 10:02, 3 March 2015 by Hesa57 (talk | contribs) (Unusable Solution)

Unusable Solution

I deleted the solution that listed out every possible number and added them up. This problem is from the AMC 12, and doing so is not a viable solution. -hesa57